Essential University Physics: Volume 1 (3rd Edition)

Published by Pearson
ISBN 10: 0321993721
ISBN 13: 978-0-32199-372-4

Chapter 14 - Exercises and Problems - Page 264: 79

Answer

You should tell her to get a 3.9 kg rope.

Work Step by Step

As we know that $\frac{v_1}{v_2}=\sqrt{\frac{F_1}{F_2}}$ $\implies \frac{v_1}{v_2}=\sqrt{\frac{m_1g}{m_2g}}$ $\implies \frac{v_1}{v_2}=\sqrt{\frac{m_1}{m_2}}$ We plug in the known values to obtain: $ \frac{18}{30}=\sqrt{\frac{1.4}{m_2}}$ This simplifies to: $m_2=3.9Kg$
Update this answer!

You can help us out by revising, improving and updating this answer.

Update this answer

After you claim an answer you’ll have 24 hours to send in a draft. An editor will review the submission and either publish your submission or provide feedback.